Find CLAT UG 2021 Solved Paper on Legal Bites. Owing to the increasing competition day by day, getting into your dream college is not that easy. Practice the CLAT UG past year paper to reflect on your preparation and increase your knowledge with the correct information. Practice makes a man perfect and thus, solving the past year paper… Read More »

Find CLAT UG 2021 Solved Paper on Legal Bites. Owing to the increasing competition day by day, getting into your dream college is not that easy. Practice the CLAT UG past year paper to reflect on your preparation and increase your knowledge with the correct information. Practice makes a man perfect and thus, solving the past year paper will provide you with an edge over your competitors. Click Here for Online Mock Tests and Solve Live.

This will allow you to grasp different concepts and assist you in developing a framework and strategy of preparation. The scores will further provide you with an analysis of your weaknesses and strengths. Attempting the paper will familiarize you with the pattern, structure and difficulty of the paper and help you ace your exams.

Find the solved CLAT PG paper below.

CLAT UG QUESTION PAPER 2021 | Click Here to Download PDF

CLAT UG QUESTION PAPER

2021

No. of questions- 150

1. Read an extract from A Scandal in Bohemia by Arthur Conan Doyle:

“I rang the door-bell and was shown up to the chamber which had formerly been in part my own. With hardly a word spoken, Sherlock Holmes waved me to an armchair. Then he stood before the fire and looked me over in his singular introspective fashion. “Watson, you did not tell me that you intended to go into harness.” “Then, how do you know?” “I see it, I deduce it. How do I know that you have been getting yourself very wet lately, and that you have a most clumsy and careless servant girl?” “My dear Holmes,” said I, “this is too much. It is true that I had a country walk on Thursday and came home in a dreadful mess, but as I have changed my clothes, I can’t imagine how you deduce it. As to Mary Jane, she is incorrigible, but there, again, I fail to see how you work it out.” “It is simplicity itself,” said he; “my eyes tell me that on the inside of your left shoe, just where the firelight strikes it, the leather is scored by six almost parallel cuts. Obviously, they have been caused by someone who has very carelessly scared round the edges of the sole in order to remove crusted mud from it. Hence, you see, my double deduction that you had been out in vile weather, and that you had a particularly malignant boot-slitting specimen of the London slavey.” In fiction, detectives like Holmes are usually portrayed as people with exceptionally brilliant minds. They possess the rare skill to see and analyze what ordinary people can’t. They have incredible abilities to infer, deduce, induce and conclude. Then, there is G.K. Chesterton’s fictional catholic priest, Father Brown who relies on his extraordinary power of sympathy and empathy that enable him to imagine and feel as criminals do. He explains, “I had thought out exactly how a thing like that could be done, and in what style or state of mind a man could really do it. And when I was quite sure that I felt exactly like the murderer myself, of course I knew who he was.” Sherlock finds the criminal by starting from the outside. He relies on science, experimental methods and deduction. On the contrary, Father Brown uses varied psychological experiences learned from those who make confessions of crime to him. He relies on introspection, intuition and empathy. There is yet another set of detectives like those created by writers like Agatha Christie. Her Belgian detective, Hercule Poirot is a story-teller who draws information from the stories that others tell. He patiently listens to numerous accounts of what happened, where it happened and how it happened. He listens for credibility and ambiguity; he identifies why and how the pieces of the jig-saws don’t fit together. Ultimately, he uncovers the truth.

From the passage, it can be inferred that:

  1. Watson is Holmes’ mentor
  2. Earlier, Watson used to live with Holmes
  3. Watson is a detective
  4. Watson shares all his personal matters with Holmes.

Answer: (b)

2. It is evident that for solving cases, Father Brown relies largely on:

  1. His own sympathetic and empathic thought process about criminals
  2. A sympathetic approach towards various people’s opinion on the case to be solved
  3. A deductive analysis of the crime and his ability to sympathize
  4. All the above.

Answer: (a)

3. For the three detectives mentioned in the passage, which one of these would be nonessential for solving criminal cases?

  1. Forgiving nature
  2. Sensitivity
  3. Critical thinking
  4. Patience.

Answer: (a)

4. In order to solve cases, Poirot uses the art of ______ the narratives that he has been told.

  1. Building a fantasy based on
  2. Empathizing with all the characters in
  3. Creating new plots for
  4. Detecting and analyzing the missing links in.

Answer: (d)

5. The word incorrigible is the antonym of:

  1. Habitual
  2. Unperformable
  3. Repentant
  4. Incurable.

Answer: (c)

Click Here for Online Mock Tests and Solve Live.

Since long, we have witnessed unimaginable levels of success and failure of various projects, businesses, scientific missions and even wars. From such triumphs and defeats emerges the much debatable thought: Is planning and strategy more important than execution? Some project leaders and their teams are of the view that planning leads to clarity of objectives; it helps to set the timeline and the budget. Consequently, when the planning is haphazard and unstructured, the very aims of the projects become hazy. This further leads to unprecedented budget collapses and poor time-management. In some cases, teams have worked relentlessly to complete assignments, but poor planning has invariably led to customer dissatisfaction and at times a complete collapse of the entire project. In the words of Benjamin Franklin, “Failing to plan is planning to fail.” Numerous entrepreneurs have the faith that strategies help to enhance not only speed and quality of production, but also consumer satisfaction. If there are no strategies to tackle unplanned events or unexpected interruptions, there is a possibility of entire projects coming to a grinding halt. Some of the world’s best airports, bridges and astronomical missions are the result of careful planning and excellent strategies. However, there are some architects, artists and entrepreneurs who prefer to dive straight from the board of ideas into the pool of execution. They believe that suitable strategies are best shaped during the process of execution; great plans and strategies can fail while encountering unexpected situations. Steve Jobs says, “To me, ideas are worth nothing unless executed. They are just a multiplier. Execution is worth millions.” According to Bill Gates, unhappy customers are the greatest source of learning. Fickle-minded consumers and wavering market trends can mar projects that stand on fixed plans. It is the need-based, flexible and innovative strategies that help to withstand the impact of these vacillating desires and trends. After rigorous planning and testing a new recipe on two lakh consumers, in 1985, the company Coke brought out the New Coke. Much to the company’s dismay, the product did not take off as expected and the financial loss was enormous. The company realized that during the process of data collection, it had not considered the product-loyalty and old-fashioned habits of the consumers. Hence, a balance of pragmatic planning, effective strategies and efficient execution is likely to ensure the accomplishment of tasks at each stage of a project. Successful execution is not an easy journey. The road is winding and bumpy. It may require tweaking or at times abandoning the original plan and re-designing it. Often, we turn to nature for inspiration. Think plans and strategies are the seed; execution is the nourishment; consumer is the capricious weather.

6. The passage can be best termed as:

  1. Expository
  2. Descriptive
  3. Abstract
  4. Narrative.

Answer: (a)

7. The author’s main purpose is to:

  1. Suggest that people succeed only when they have great strategies
  2. Highlight the fact that execution is as indispensable as planning and strategy
  3. Illustrate the impact of poor planning and weak strategy
  4. Highlight the hurdles that come in the way of execution.

Answer: (b)

8. From the failure of New Coke, the company learnt that:

  1. Tested and tried strategies are essential for success
  2. Surveys and data calibration are a sheer waste of time
  3. Consumers change brand preferences too frequently
  4. Changes in consumers’ habits should not be taken for granted.

Answer: (d)

9. For the accomplishment of a project, Steve Jobs:

  1. Propagates an approach that is similar to Benjamin Franklin’s
  2. Suggests an approach that is different from Benjamin Franklin’s
  3. Believes that the main control is in the hands of consumers
  4. None of the above.

Answer: (b)

10. The ______ in ‘consumer is the capricious weather’ refers to the ______ demands of the customers.

  1. Onomatopoeia ……….. stagnant
  2. Simile …………. unpredictable
  3. Metaphor……….. wavering
  4. Anaphora………oscillating.

Answer: (c)

Since the worldwide inoculation process is going strong, vaccine diplomacy has become a hot topic. In their quest for ensuring vaccine security, a report by The New York Times, based on the data on vaccine contracts compiled by Duke University, shows that the advance purchase contracts made by some advanced countries for potential vaccines would vaccinate their population many times: the European Union, two times, the United States and the United Kingdom, four times, and Canada, six times. The expectation that an early vaccination will bring back normalcy and a required push to economic growth fuelled many advanced countries to engage in vaccine battles. The arguments of public good and global cooperation have gone out of the window now. While advanced countries have turned their back on the need of poor countries to access COVID-19 vaccines, India has displayed empathy to their needs. India has taken a position that a significant percentage of the approved doses will be permitted for exports. While its exports to neighbouring countries will be under grant mode, initial shipment of vaccines to least developed countries will be free of cost. And, shipments of vaccines from India have already started reaching different parts of the developing world. While India is in its first phase of vaccination to cover health-care workers, exports from India are helping other countries also in initiating phase one of their vaccination programme, a gesture well appreciated globally. In a democracy, one can expect the backlash of sending vaccines abroad without vaccinating its population. Nevertheless, India’s approach only reinforces the need of having coordinated global efforts in bringing COVID-19 under control. This response manifests India’s unstinted commitment to global development and has consolidated its name as the world’s pharmacy. The attitude of India towards vaccinating the populations in the poorer countries has generated discussion in the richer countries about the necessity for more proactive measures to roll out vaccines to the developing nations.

11. Which of the following best describes the purpose of this passage?

  1. To encourage vaccine nationalism, and discourage global cooperation
  2. To discourage vaccine nationalism, and encourage global cooperation
  3. To encourage poor countries and discourage advanced countries
  4. To encourage India to provide vaccines to poor nations.

Answer: (b)

12. The term ‘inoculation process’ as used in the passage means:

  1. Production and distribution of vaccines
  2. Global struggle for vaccines
  3. Pharmaceutical production of vaccines
  4. Artificially inducing immunity.

Answer: (d)

13. Advanced purchase contracts of vaccines by developed countries are premised on hopes that:

  1. Vaccination will restore normalcy and thrust economic recovery
  2. Vaccination will bring back powers to nations
  3. Vaccination will promote global well-being and growth
  4. Vaccination will protect the health of the world population.

Answer: (a)

14. The author cautions that India’s approach in vaccine distribution may have negative repercussions among:

  1. People from advanced countries
  2. People from poor countries
  3. People from developing countries
  4. People from India.

Answer: (d)

15. Among Sustainable Development Goals (SDGs), the third goal reads, “Ensure healthy lives and promote well-being for all at all ages”. Which of the following sentences from the passage reflects the reversal of this SDG?

  1. The arguments of public good and global cooperation have gone out of the window now
  2. Advanced countries have turned their back on the need of poor countries to access COVID-19 vaccines
  3. The advance purchase contracts made by some advanced countries for potential vaccines would vaccinate their population many times
  4. All the above.

Answer: (d)

Following the transition to democracy, with the inauguration of Nelson Mandela as president in I994, South Africa was faced with the task of dealing with its past, as well as undertaking some action to deal with structural social injustice. The Truth and Reconciliation Commission (TRC), heralded as the most ambitious and organised attempt to deal with crimes of a past regime through a concept of truth, came into force on 19th July I995 in South Africa. Emerging as a political strategy to acknowledge past suffering whilst promoting a future based on the concerns of social justice, the rule of law and reconciliation, the Commission has struggled to fulfil its objectives. Although the TRC incorporated these broader concerns into the mandate of its three sub-committees, they were disregarded in practice. These sub-committees, which reflected concerns for ‘human rights violations’, ‘amnesty’ and ‘reparation and rehabilitation’, were not ‘coupled with some form of social transformation’. The public transition from apartheid, established through a negotiated settlement rather than a revolutionary process, framed the Commission’s powers. Shaped by the historical context of this particular transition, the TRC was careful not to ‘rock the structural boat’. Rather than pursuing truth and justice, as an integrated feature of social transformation, the Commissioners and, to a greater extent, the government of South Africa, maintained an agenda that avoided a challenge to the status quo. A focus on restorative justice was taken by the Commission with an emphasis placed on mechanisms to restore victims and survivors, through reparations policy, state-led acknowledgement of suffering, and a condemnation, together with the transformation, of the system that implemented such widespread forms of abuse. The priority of changing the apartheid conditions of gross inequality and oppression provided a backdrop to the approval of the TRC by those who had suffered. More difficult to accept was the provision of amnesty to those who had undertaken violations of human rights. The process placed amnesty of violations as a carrot to perpetrators in exchange for a full story, with the stick of prosecutions for those who did not come forward.

16. ‘The status quo’ as used in the passage means:

  1. Previously popular opinions
  2. Already existing conditions
  3. Strategies of government
  4. Following a set agenda.

Answer: (b)

17. Which of the following best describes the tone of the author?

  1. Optimistic
  2. Threatening
  3. Compassionate
  4. Critical.

Answer: (d)

18. Which of the following is not the broader concern of Truth and Reconciliation Commission (TRC)?

  1. To deal with crimes of a past regime through a concept of truth
  2. To acknowledge past suffering
  3. To emerge as a political strategy for reconciliation of rule of law
  4. To promote a future based on the concerns of social justice.

Answer: (c)

19. Which of the following statements is least likely to be inferred from the passage?

  1. The TRC ignored some of the broad concerns
  2. The sub-committees of TRC lacked an agenda of social transformation
  3. The TRC made earnest efforts to protect human rights
  4. The TRC lacked the desire and strength to challenge the prevailing conditions.

Answer: (c)

20. What does ‘amnesty’ mean in the context of the passage?

  1. Severe punishment
  2. Bring legal action
  3. Arrest warrants
  4. Official pardon.

Answer: (d)

In the domain of education, the current pandemic has made three things clear. It has proved beyond any doubt that we need schools. Irrespective of which country one talks about, students and parents want schools to open and function in full glory, with appropriate precautions. Secondly, it has shown that technology may prove to be useful in education if it is employed thoughtfully. Random surfing of the Internet may lead to a collection of pieces of information that do not add up to any meaning. As Noam Chomsky says, “You cannot pursue any kind of inquiry without a relatively clear framework that is directing your search and helping you choose what is significant and what is not.” Moreover, there cannot be any hegemonic techno managerial solutions to the linguistic and cultural heterogeneity of students; technology must help us to respect individual, peer group and community needs and aspirations. Thirdly, a convergence of the efforts of the public, civil society and private enterprise will have to take place if we wish technology to meaningfully mediate between school and home, particularly among underprivileged groups. The concept, structure and functioning of a school/college should not be trivialised in any way. This institution has survived since ancient times in spite of proposals for “de -schooling” of various kinds. It is true that schools to a great extent perpetuate the status quo and, as Ivan Illich observed, encourage “consumerism” and “obedience to authority”; but it is also true that those who produced some of the most revolutionary moments in history, including quantum jumps in knowledge, also went to school. The kind of web of learners Illich imagines may in fact have its roots in schools. There are also people who trivialise schools for the kind of investments they demand in terms of space, buildings, teachers, libraries and labs and other infrastructure. One thing you never forget is the school you went to, friends you made there and the kind of teachers who taught you; the kind of teachers you loved, the kind you mocked at with friends. You recollect nostalgically the sports and other co-curricular activities you took part in. Some of you may still have preserved your school blazer, trophies and photographs with a sense of joy. It is important to see school holistically; it is not a set of atomic items of rooms, library, assembly halls, canteen and playgrounds; it is all of these but in symbiotic relationship with each other, the contours of which are often far too obvious and often simply mysterious.

21. Which one of the following is the author trying to suggest by quoting Noam Chomsky?

  1. Technology shall be used in purposively structured manner in education
  2. A relatively clear framework pursues any kind of inquiry
  3. Technology helps to choose what is significant and reject what is not significant
  4. Technology must help us to respect individual, peer group and community needs and aspirations.

Answer: (a)

22. Which of the following is not a premise of author’s argument in favour of need for schools and colleges?

  1. Schools and colleges provide opportunities for socialization
  2. The institution of schools and colleges has shown perseverance
  3. The current pandemic has proved that students and parents want schools and colleges
  4. Random surfing of the internet may lead to a collection of meaningless information.

Answer: (d)

23. The expression ‘linguistic and cultural heterogeneity’ as used in the passage means:

  1. Linguistic and cultural unity
  2. Linguistic and cultural unity in diversity
  3. Linguistic and cultural diversity
  4. Linguistic and cultural identity.

Answer: (c)

24. Which of the following is not the learning outcome from pandemic?

  1. Parents and students from around the world need schools
  2. Technology is undoubtedly and absolutely useful in education
  3. Technology may be used meaningfully to mediate between schools and home among underprivileged groups
  4. None of the above.

Answer: (b)

25. The expression ‘symbiotic relationship’ as used in the passage means:

  1. Mutually inclusive relationship
  2. Mutually exclusive relationship
  3. Mutually beneficial relationship
  4. Both (b) and (c).

Answer: (c)

A new report forecasting that India can create millions more jobs over the coming years in thegig economy underscores a fundamental shift in the nature of work. While automation swept through factory floors and BPOs reduced manpower requirements, e-commerce, ride hailing and food delivery apps, streaming media and fintech have created lakhs of temporary jobs in the services sector. Although the jury is still out on the quality of life accorded by such gigs and the social security benefits they accord, recall that informalisation of jobs started much earlier. The report by Boston Consulting Group and Michael & Susan Dell Foundation predicts 90 million flexi and gig jobs in a decade from 8 million now, contributing transactions valued at more than $250 billion and an additional 1.25% to India’s GDP. Obviously, technological evolutions are hard to predict. A decade ago, few, if any, had divined these new jobs. Accepting change as the sole constant, it is equally critical to create the socio-economic framework that can support such jobs. Expecting startups fuelled by venture capital and presently unprofitable to treat gig workers as regular employees isn’t practical. Such moves could impede innovation and investment. But if central and state governments could deliver in areas like public health, education, insurance and food security, anxieties generated by unsteady, irregular unemployment can be managed better. Last year’s nationwide lockdown when the suddenly unemployed migrant workers panicked and bolted, untrusting the promises of governments to care for them, served out this lesson in poignant detail. The gig economy does promise flexibility and improved choices for many women and part time workers. Every technological revolution till date has effaced some jobs and created plentiful others. There is room for optimism in the rapidly emerging tech mediated world, but only upon strengthening the support of educational and health infrastructure.

26. The report by Boston Consulting Group and Michael & Susan Dell Foundation ______

  1. Predicts massive rise in gig jobs in a decade
  2. Predicts fundamental shift in nature of work
  3. Predicts massive rise in World’s GDP
  4. All the above.

Answer: (a)

27. Which of the following can be inferred as a caution by the author?

  1. Central and state governments must ensure management of public health, education, insurance and food security, anxieties of gig workers
  2. Expecting treatment of gig workers as regular employees could impede innovation and investment
  3. If educational and health infrastructure support is not strengthened, gig economy will fail
  4. It is critical to create the socio-political framework that can support gig jobs.

Answer: (b)

28. In the context of the passage, which of the following best describes gig economy?

  1. Informally structured economy with skill based short-term or freelance labour market
  2. A fundamental shift in the nature of work in an economy
  3. Economy characterized by startups fuelled by venture capital
  4. All the above.

Answer: (a)

29. The tone of the passage is:

  1. Cynical
  2. Critical
  3. Scornful
  4. Sanguine.

Answer: (d)

30. According to the passage, which of the following comes as a concern related to gig economy?

  1. Lack of educational and health infrastructure
  2. Temporary nature of work
  3. The quality of life and the social security
  4. None of the above.

Answer: (c)

Current Affairs and General Knowledge Starting in April 2020, China amassed a large number of troops and armaments along the disputed Line of Actual Control (LAC) in eastern Ladakh and other areas along the LAC, leading to stand-offs and skirmishes at certain points. The People’s Liberation Army (PLA) moved into Indian territory and built fortified structures and defences, changing the status quo on the ground. While there are differences in perception on the alignment of the LAC, over the years both sides concluded a series of agreements to maintain peace and tranquillity on the border, while talks continued to resolve the dispute. Since then, several friction points emerged in eastern Ladakh and both sides initiated diplomatic and military talks aimed at disengagement and de-escalation, with India pushing for the restoration of status quo ante of pre-April. India and China earlier reached an understanding for phased disengagement from all friction points followed by de-escalation from the depth areas along the LAC.

31. Violent clashes between Indian and Chinese army which resulted in death of around 20 Indian soldiers and an unknown number of Chinese soldiers occurred on:

  1. May 5, 2020
  2. June 11-12, 2020
  3. June 15-16, 2020
  4. July 13, 2020.

Answer: (c)

32. Where did the violent clashes between Indian and Chinese soldiers occur which caused casualties on both sides?

  1. Naku La
  2. Dhan Singh Thapa post
  3. Galwan Valley
  4. Siachen.

Answer: (c)

33. The violent incident leading to death of soldiers took place at:

  1. Patrol Point 13
  2. Patrol Point 14
  3. Patrol Point 15
  4. Patrol Point 16.

Answer: (b)

34. The length of Line of Actual Control of Indo-China Border is:

  1. 2788 km
  2. 3213 km
  3. 3317 km
  4. 3488 km.

Answer: (d)

35. Which of the following was resolved by way of an agreement between India and China in February 2021?

  1. Chinese side will move back its troops to east of finger 8 and Indian side will move its troops to its permanent base near finger 3
  2. Both sides shall demolish structures built after April 2020
  3. Moratorium on patrolling in the affected area
  4. All of these.

Answer: (d)

As soon as the coronavirus hit the world, India emphasised the imperative of collaboration to overcome the inimical health and economic impact of the virus. It was with this objective that the Indian government organised a virtual meeting of SAARC leaders on March 15, 2020. In the initial months of the pandemic, India, by virtue of the fact that it is the ‘pharmacy of the world’, ramped up production of essential medicines like hydroxychloroquine, paracetamol etc. as well as Personal Protection Equipment (PPE) kits, ventilators and masks. In the rapidly evolving global geo-political landscape, the healing and supportive actions by India through supply of Covishield and Covaxin vaccines will have a huge impact in promoting peace, security, cooperation and prosperity in the region and the world. This is all the more commendable when there are only few countries in the world who have thus far been able to successfully manufacture COVID-19 vaccines.

36. The Minister of Health and Family Welfare is:

  1. Shri Mansukh Mandaviya
  2. Harsh Vardhan
  3. Shri Ashwini Kumar Choubey
  4. Shri Rajnath Singh.

Answer: (a)

37. What is the amount that India has created to contribute in SAARC COVID-19 Emergency Fund?

  1. $20 million
  2. $10 million
  3. $15 million
  4. $17 million.

Answer: (b)

38. Which of the following countries were the first to receive supplies of vaccines from India?

  1. Nepal and Bangladesh
  2. Bhutan and Myanmar
  3. Bangladesh and Bhutan
  4. Bhutan and Maldives.

Answer: (d)

39. Under which of the following initiatives has the Government of India decided to supply vaccines to the foreign countries?

  1. Vaccine Ujala
  2. Vaccine Dosti
  3. Vaccine Maitri
  4. Vaccine Rishta.

Answer: (c)

40. COVAXIN, India’s indigenous COVID-19 vaccine is developed by_________

  1. Indian Council of Medical Research (ICMR) in partnership with Bharat Biotech International Limited (BBIL)
  2. Indian Council of Medical Research (ICMR)
  3. Bharat Biotech International Limited (BBIL)
  4. Serum Institute of India.

Answer: (a)

41. There is really nothing new, not already anticipated in a distressed planet since the emergence of human species life nearly two million years ago. So, a long prehistory of inter-faith dialogue entailing the reciprocal relation between religious traditions and the robustness of the very idea of being human and having rights, precedes the recent UN movement of faith for rights (F4R). The F4R framework has finally been affirmed by the United Nations system. The Office of the High Commissioner for Human Rights (OCHR) promoted the Rabat plan of action in 2012. Note that this happened after the end of the Cold War, during globalisation, and soon after 9/11. The plan was a result of a series of expert workshops on the prohibition of incitement to national, racial or religious hatred, underlying “legislative patterns, judicial practices and policies”. The UN Human Rights Council is shortly going to discuss further the prohibition of the advocacy of national, racial or religious hatred that constitutes incitement to “discrimination, hostility or violence”.

41. When was the first human rights declaration adopted by the United Nations?

  1. December 10, 1945
  2. September 10, 1945
  3. December 10, 1948
  4. September 10, 1948.

Answer: (c)

42. The United Nations Human Rights Council is a/an __________ body within the United Nations System.

  1. Inter-governmental
  2. Non-governmental
  3. Quasi-governmental
  4. Multi-stakeholder.

Answer: (a)

43. The Universal Declaration of Human Rights (UDHR), the International Covenant on Economic, Social and Cultural Rights (ICESCR), and the International Covenant on Civil and Political Rights (ICCPR) and its two Optional Protocols have been adopted under the auspices of the United Nations and are collectively referred to as____________

  1. Magna Carta Libertatum
  2. International Bill of Human Rights
  3. International Charter of the Rights of the Man and the Citizen
  4. International Charter of Human Rights.

Answer: (b)

44. The objective of the ‘Faith for Rights’ (F4R) is to provide space for a cross-disciplinary reflection and action on the deep, and mutually enriching, connections between religions and human rights. Which of the following is not one of the commitments on Faith for Rights (F4R)?

  1. To prevent the use of the notion of ‘State religion’ to discriminate against any individual or group
  2. To revisit religious interpretations that appear to perpetuate gender inequality and harmful stereotypes or even condone gender-based violence
  3. To refine the curriculums, teaching materials and textbooks to rectify harmful stereotypes
  4. To promote religious dogmatism.

Answer: (d)

45. In India, which of the following statutory organisations is responsible for the protection and promotion of human rights?

  1. National Human Rights Council
  2. National Human Rights Commission
  3. National Human Rights Organisation
  4. Indian Human Rights Council.

Answer: (b)

UNESCO New Delhi, along with the National Mission for Clean Ganga (NMCG) and other partners, organised a hybrid event with school children and teachers to celebrate ‘World Water Day 2021’. The event included felicitating the winners of a water conservation awareness programme, and screening of their winning animation short films, according to an official release. UNESCO launched this nation-wide programme for school children in India, in partnership with the National Mission for Clean Ganga, the United Schools Organization (USO), Water Digest and India based global animation major Toonz Media Group. Entitled ‘H2Ooooh!-Waterwise program for children of India’, this innovative initiative encouraged school students between the age of 6-14 years to submit story ideas for animated short films to raise awareness on water conservation and its sustainable use.

46. World Water Day is observed on:

  1. April 20
  2. April 21
  3. March 22
  4. May 20.

Answer: (c)

47. The theme of World Water Day 2021 was:

  1. Valuing Water
  2. Conserving Water
  3. Saving Water
  4. Drinking Water.

Answer: (a)

48. On the occasion of World Water Day 2021, Prime Minister Narendra Modi launched a campaign named:

  1. Jal Shakti Abhiyan: Purify the Rivers
  2. Jal Shakti Abhiyan: Catch the Rain
  3. Jal Shakti Abhiyan: No More Water Woes
  4. Jal Shakti Abhiyan: Become Water Wise.

Answer: (b)

49. The National Water Mission (NWM) has been constituted under the National Action Plan for Climate Change (NAPCC) launched by the Prime Minister of India in 2009. Which of the following is not a goal of the NWM?

  1. Comprehensive water data base in public domain and assessment of the impact of climate change on water resource
  2. Promotion of citizen and state actions for water conservation, augmentation and preservation
  3. Comprehensive water data base in public domain and assessment of the impact of water resource on climate change
  4. Increasing water use efficiency by 20%.

Answer: (c)

50. In 2019, the Ministry of Jal Shakti was formed after merging the Ministry of Drinking Water and Sanitation and the Ministry of ______

  1. Water Resources, River Development and Ganga Rejuvenation
  2. Ports, Shipping and Waterways
  3. Water Conservation and Ganga Rejuvenation
  4. Inland Waterways and Ganga Rejuvenation.

Answer: (a)

Click Here for Online Mock Tests and Solve Live

Prime Minister Narendra Modi, during his visit to Bangladesh, congratulated the neighbouring country for its 50th year of Independence and said that he too had gone to jail while protesting for Bangladesh’s liberation. “I was about 20-22 years old when along with some friends I had protested in favour of Bangladesh during its struggle. I even went to jail. Pakistan’s atrocities in Bangladesh are well known…the pictures didn’t allow me to sleep,” PM Modi said. He also paid respect to the Indian Army who fought for the freedom of Bangladesh and also paid respect to Bangladesh’s Father of the Nation, ‘Bangabandhu’ Sheikh Mujibur Rahman and conferred upon him the Gandhi Peace Prize 2020. “I pay my respects to Bangabandhu. He gave his life for the people of Bangladesh. Bangabandhu Mujibur Rehman was a ray of hope. He made sure that no country could enslave Bangladesh.”, PM Modi said.

51. The Gandhi Peace Prize is awarded for:

  1. Non-cooperation against social injustice
  2. Humanitarian leadership
  3. Social and economic development in villages
  4. Social, economic and political transformation through non-violence.

Answer: (d)

52. The Gandhi Peace Prize was awarded for the first time in the year 1995 to:

  1. Julius K. Nyerere, former President of Tanzania
  2. Archbishop Desmond Tutu, South Africa
  3. John Hume, Ireland
  4. Baba Amte (Murlidhar Devidas Amte), India.

Answer: (a)

53. Which of the following Indian organisations was awarded the Gandhi Peace Prize in 2014?

  1. Defence Research and Development Organisation (DRDO)
  2. Bhabha Atomic Research Center (BARC)
  3. Indian Space Research Organization (ISRO)
  4. Radha Soami Satsang Beas (RSSB).

Answer: (c)

54. Sheikh Mujibur Rahman served as the first ______ of Bangladesh.

  1. Prime Minister
  2. President
  3. Home Minister
  4. Defence Minister.

Answer: (b)

55. Sheikh Mujibur Rahman was assassinated on ______

  1. August 15, 1960
  2. November 15, 1975
  3. August 15, 1975
  4. November 15, 1960.

Answer: (c)

The search for efficient trade routes and better connectivity has been one of the powerful driving forces of history. It has determined the prospects of many nations, regions and civilizations over time. Today, as the world goes through a fundamental re-balancing, correcting many of the distortions of the past, it is only natural that connectivity should be central to that exercise. Growth in trade, commerce, industrial development and technological advancement has gone hand in hand with ease of connecting. Maritime connectivity, in particular, has played a significant role in creating regional corridors for trade and economic linkages, as indeed for cultural and intellectual exchanges over the centuries. The shift in the fulcrum of global economic growth towards Asia is creating unprecedented opportunities for connectivity in the region. The Government of India, recognising the importance of regional connectivity, made a landmark decision to undertake an overseas port investment in Chabahar.

56. The Chabahar Day was commemorated by the Ministry of Ports, Shipping and Waterways in which of the following Summit?

  1. India Water Impact Summit
  2. Sustainable Ocean Summit
  3. World Ocean Summit
  4. Maritime India Summit.

Answer: (d)

57. Which of the following is correct about the Chabahar Port?

  1. It is located in south east of Iran in Oman sea
  2. It has marine border of more than 300 km
  3. Both (a) and (b)
  4. None of the above.

Answer: (c)

58. Which of the following statements relate to the Chabahar Port and its impact?

  1. It is a transit hub to support landlocked countries of Central Asia
  2. It will play an important role in international North-South transit Corridor (INSTC)
  3. It will optimize the logistic cost by bringing in efficiency, and creating a reliable and safe transport corridor
  4. All of the above.

Answer: (d)

59. The Chabahar Port is a result of trilateral agreement between which of the following countries?

  1. India-Iran-Afghanistan
  2. India-Iran-Iraq
  3. Iran-Iraq-Afghanistan
  4. India-Qatar-Pakistan.

Answer: (a)

60. Which is the Special Purpose Vehicle (SPV) incorporated for the development and management of Shahid Beheshti Port of Chabahar?

  1. The Indian Port Rail Corporation
  2. Western Shipbuilding Private Limited
  3. India Ports Global Limited
  4. Karaikal Port Private Limited.

Answer: (c)

India has added Tso Kar Wetland Complex in Ladakh as its 42ndRamsar site, which is a second one in the Union Territory (UT) of Ladakh. Expressing happiness, Minister for Environment, Forest and Climate Change, Shri Prakash Javadekar shared this information in a tweet message. The Tso Kar Basin is a high-altitude wetland complex, consisting of two principal waterbodies, Startsapuk Tso, a freshwater lake of about 438 hectares to the south, and Tso Kar itself, a hypersaline lake of 1800 hectares to the north, situated in the Changthang region of Ladakh, India. It is called Tso Kar, meaning white lake, because of the white salt efflorescence found on the margins due to the evaporation of highly saline water. The Tso Kar Basin is an A1 Category Important Bird Area (IBA) as per Bird Life International and a key staging site in the Central Asian Flyway. The site is also one of the most important breeding areas of the Blacknecked Crane (Grus nigricollis) in India. This IBA is also the major breeding area for Great Crested Grebe (Podicepscristatus), Bar-headed Geese (Anserindicus), Ruddy Shelduck (Tadornaferruginea), Brown-headed Gull (Larusbrunnicephalus), Lesser Sand-Plover (Charadriusmongolus) and many other species.

61. Which of the following sites in India is not included in the Ramsar Wetland List?

  1. Asan Conservation Reserve, Uttarakhand
  2. Dal Lake, Jammu and Kashmir
  3. Kabartal Wetland, Bihar
  4. Lonar Lake, Maharashtra.

Answer: (b)

62. The World Wetlands Day is celebrated on:

  1. February 15
  2. February 6
  3. February 2
  4. February 28.

Answer: (c)

63. The theme for World Wetlands Day 2021 was:

  1. Wetlands and Climate Change
  2. Wetlands and Water
  3. Wetlands and Biodiversity
  4. Wetlands for our Future: Sustainable Livelihoods.

Answer: (b)

64. The World’s largest mangrove forest is located in:

  1. Sundarban Mangrove
  2. Pichavaram Mangrove
  3. Florida Mangrove
  4. Godavari-Krishna Mangroves.

Answer: (a)

65. On the occasion of the World Wetland Day, 2021 and as a part of commitment towards conservation, restoration and management of India’s wetlands, establishment of a centre was announced by the name of:

  1. Centre for Wetland and Coastal Management
  2. Centre for Wildlife Conservation and Management
  3. Centre for Wildlife Management
  4. Centre for Wetland Conservation and Management.

Answer: (d)

Legal Reasoning Harm suffered voluntarily does not constitute a legal injury and is not actionable. This principle is embodied in the maxim volenti non fit injuria. A person cannot complain of harm to the chances of which he has exposed himself with his free consent and free will. The maxim volenti non fit injuria is founded on good sense and justice. A person who has invited or assented to an act being done towards him cannot, when he suffers from it, complain of it as a wrong. The maxim presupposes a tortious act by the defendant. The maxim applies, in the first place, to intentional acts which would otherwise be tortious. There are certain limitations to the application of this maxim: (i) It is no answer to a claim made by a workman against his employer for injury caused through a breach by the employer of a duty imposed upon him by a statute. But where the negligence or breach of statutory duty is on the part of an employee of the plaintiff who knowingly accepts the risk flowing from such breach and the employer-defendant is not guilty of negligence or breach of statutory duty, the defence of volenti non fit injuria is available to the defendant. (ii) Under an exigency caused by the defendant’s wrongful misconduct, consciously and deliberately faced a risk, even of death, whether the person endangered is one to whom he owes a duty of protection, as a member of his family, or is a mere stranger to whom he owes no such special duty. The rescuer will not be deprived of his remedy merely because the risk which he runs is not the same as that run by the person whom he rescues. But where there is no need to take any risk, the person suffering harm in doing so cannot recover. (iii) To cover a case of negligence the defence on the basis of the maxim must be based on implied agreement whether amounting to contract or not. The defence is available only when the plaintiff freely and voluntarily, with full knowledge of the nature and extent of the risk impliedly agreed to incur it and to waive any claim for injury. But when the plaintiff has no choice or when the notice is given at a stage when it is beyond the ability of the plaintiff to make a choice there can be no implied agreement and the defence on the basis of the maxim must fail. (iv) The maxim will also not apply when the act relied upon is done because of the psychological condition which the defendant’s breach of duty had induced.

66. Mr. A was the owner of a car and he had a driver- Mr. D. On January 19, 2021, Mr. A and Mr. D were travelling in their car wherein Mr. A got down at a restaurant and told Mr. D to take the car back to Mr. A’s bungalow. Mr. D was filling the petrol tank of the car, and two strangers- Mr. B and Mr. C took a lift from Mr. D in his car. The car went ahead and the right-side front wheel of the car flew away, the car toppled and Mr. D and Mr. C were thrown out. Mr. C sustained severe injuries and ultimately died due to those injuries on January 20, 2021. Mr. B and legal representatives of Mr. C claimed compensation from Mr. A and Mr. D.

  1. D will be liable to pay the compensation
  2. Volenti non fit injuria will be applicable and no compensation can be claimed
  3. Volenti non fit injuria will not be applicable and compensation can be claimed
  4. A and Mr. D both will be liable to pay the compensation.

Answer: (b)

67. Rama was a spectator at a motor car race being held on a track owned by the defendant company. During the race, there was a collision between two cars, one of the cars was thrown among the spectators, thereby injuring Rama severely. Which of the following statements is correct?

  1. Rama impliedly took the risk of such injury, the danger being inherent in the sport which any spectator could foresee, the defendant was not liable
  2. It was a negligence on the part of defendant and volenti non fit injuria will be applicable
  3. Rama did not take the risk of such injury, and she only consented to watching the race and hence the defendant was liable
  4. Rama was negligent and hence she suffered injuries.

Answer: (a)

68. Which of the following is correct about consent in volenti non fit injuria?

  1. Knowledge of the risk does not always amount to consent
  2. Knowledge of a risk does not precede consent
  3. Knowledge of the risk always amounts to consent
  4. Mere perception of the existence of danger amounts to consent.

Answer: (a)

69. Lily had placed spring guns in a wood on her ground for the protection of the garden. Karan, with full knowledge that there were spring guns somewhere in the wood, trespassed on the land of Lily and was injured. Which of the following statements is correct?

  1. Lily will be liable to pay compensation to Karan
  2. Lily has not committed a tort against Karan by exceeding her right of private defence
  3. Karan’s case does not fall within volenti non fit injuria
  4. Karan had knowledge of the spring guns and wilfully courted the danger himself.

Answer: (d)

70. Which of the following is not an element to claim the defence of volenti non fit Injuria?

  1. Prior knowledge of the plaintiff about the risk involved
  2. Free consent
  3. Plaintiff is compelled to agree to a risk by the defendant
  4. Voluntary acceptance of the risk by the plaintiff.

Answer: (c)

It is essential to the creation of a contract that both parties should agree to the same thing in the same sense. Mutual consent, which should also be a free consent, is the sine qua non of a valid agreement and one of its essential elements is that a thing is understood in the same sense by a party as is understood by the other. Not only consent, but free consent is provided in Section 10 of the Indian Contract Act, 1872 to be necessary to the complete validity of a contract. Consent is free when it works without obstacles to impede its exercise. Where there is no consent or no real and certain object of consent, there can be no contract at all. Where there is consent, but not free consent, there is generally a contract voidable at the option of the party whose consent was not free. A general averment that consent was not freely obtained is not enough, and it is necessary to set up one of the vitiating elements such as fraud which includes, false assertion, active concealment, promise without intention of performing it, any other deceptive act, or any act declared as fraudulent. In order to constitute fraud, the act should have been done by the party to the contract, or by any other person with his connivance, or by his agent and with intent to deceive the other party thereto or his agent, or to induce him to enter into the contract. There is no duty upon parties to speak about facts likely to affect the other party’s consent to the contract and mere silence does not amount to fraud, unless the circumstances of the case show that there is duty to speak, or silence is, in itself equivalent to speech. On the other hand, misrepresentation falls into three categories: (i) a statement of fact, which if false, would be misrepresentation if the maker believes it to be true, but which is not justified by the information he possesses; (ii) any breach of duty which gains an advantage to the person committing it by misleading another to his prejudice, there being no intention to deceive; and (iii) causing a party to an agreement to make a mistake as to the substance of the thing which is the subject of the agreement, even though done innocently.

71. Which of the following statements correctly depicts the essentials of misrepresentation?

  1. A misrepresentation is a positive statement of fact, which is made or adopted by a party to a contract and is untrue
  2. Misrepresentation and false representation do not mean the same
  3. If one party has induced the other to enter into a contract by misrepresenting, though innocently, any material fact especially within his own knowledge, the party misled cannot avoid the contract
  4. A misrepresentation is a negative statement of fact, which is made or adopted by a party to a contract and is true.

Answer: (a)

72. Consider the statements given below and answer which one correctly describes a fraudulent act.

(I) The expression fraud means an intention to deceive, whether it is from any expectation of advantage to the party himself or from ill will towards the other is immaterial. (II) A fraud is an act of deliberate deception with the design of securing something by taking an unfair advantage of another. It is a deception to gain from another’s loss. (III) Fraud arises out of deliberate active role of representator about a fact.

  1. (I), (II) are correct
  2. (I) correct
  3. (I), (II), (III) are correct
  4. (I) and (II) are correct but (III) is incorrect.

Answer: (c)

73. Which of the following statements is correct?

  1. Fraud is an innocent wrong whereas misrepresentation is an intentional wrong
  2. The principal difference between fraud and misrepresentation is that in the former, the person making the suggestion does not believe it to be true and, in the latter, he believes it to be true
  3. In fraud and misrepresentation both, it is not a misstatement of fact which misleads the promisee
  4. Fraud and misrepresentation both are innocent wrongs.

Answer: (b)

74. Mr. A sells a car to Mr. Y, his childhood friend with a knowledge that the car is defective. Before buying the car, Mr. Y says to Mr. A, “If you do not deny it, I shall assume that the car is perfect”. Mr. A says nothing. In light of the statement, decide the liability of Mr. A.

  1. A’s silence is equivalent to speech and hence a misrepresentation
  2. A is not liable for fraud, but liable for misrepresentation
  3. A is liable for fraud and misrepresentation both
  4. A’s silence is equivalent to speech and hence a fraud.

Answer: (d)

75. In which of the following statements will a contract not be voidable at the option of a party?

  1. When a party takes consent by fraud
  2. When a party takes consent by misrepresentation
  3. A contract entered by fraud and misrepresentation is neither void nor voidable
  4. When silence amounts to fraud, but the other party whose consent was taken had discovered the truth or had the means of discovering the truth with ordinary diligence.

Answer: (d)

Section 4 of the Indian Contract Act, 1872 reads as follows: Communication when complete – The communication of a proposal is complete when it comes to the knowledge of the person to whom it is made. The communication of an acceptance is complete, – as against the proposer, when it is put in a course of transmission to him so as to be out of the power of the acceptor; as against the acceptor, when it comes to the knowledge of the proposer. Thus, the provision makes no difference in the position of the offeror. The offeror becomes bound when a properly addressed and adequately stamped letter of acceptance is posted. The acceptor does not become bound by merely posting his acceptance. He becomes bound only when his acceptance comes to the knowledge of the proposer. The contract is concluded at the place from where the proposal is accepted and communication of acceptance is dispatched, i.e., the address at which the proposal was sent. The court at that place would have jurisdiction to entertain a cause of action under the contract. This rule, that the communication of an acceptance is complete as against the proposer when the letter is posted, is probably intended to apply only when the parties are at a distance and they communicate by post. “Where, however, the parties are in each other’s presence or, though separated in space”, they are in direct communication, as, for example, by telephone, no contract will arise until the offeror receives the notification of acceptance.

76. ‘S’ wanted to purchase shares of a company and communicated his offer to buy shares on March 1, 2021. A letter of allotment of shares addressed to ‘S’, which is an acceptance of the offer to purchase shares, was posted by the company on March 10, 2021, but the letter never reached ‘S’ and was lost in transit. In the given situation, which of the following statements is true?

  1. Communication of acceptance is not complete as against ‘S’ and hence, there is no valid contract between ‘S’ and the company
  2. Communication of acceptance is complete as against ‘S’ however not complete as against the company
  3. Communication of acceptance is complete as against the company however not complete as against ‘S’
  4. Communication of acceptance is complete against both ‘S’ as well as the company.

Answer: (b)

77. ‘A’, who is in Mumbai, makes an offer for supply of goods to ‘B’, who is in Delhi, via a mobile phone call. During the same call, A’s offer is absolutely and unconditionally accepted by ‘B’. According to the terms agreed between ‘A’ and ‘B’, goods are to be supplied at Pune and payment is to be made electronically. In the given situation, where is the contract concluded?

  1. Neither Mumbai, Delhi nor Pune as it is a telephonic contract
  2. Pune
  3. Delhi
  4. Mumbai.

Answer: (d)

78. ‘X’, who is in Gandhinagar, makes an offer for sale of second-hand luxury car to ‘Y’, who is Jammu, via an e-mail sent on January 15, 2021 at 2:03 pm. X’s offer is absolutely and unconditionally accepted by ‘Y’ via an e-mail sent on January 15, 2021 at 4:04 pm. The e-mail communicating acceptance is read by ‘X’ on January 15, 2021 at 7:05 pm. In the given situation, when is the contract concluded?

  1. As against ‘X’, on January 15, 2021 at 4:04 pm and as against ‘Y’, on January 15, 2021 at 7:05 p.m.
  2. As against ‘Y’, on January 15, 2021 at 4:04 pm and as against ‘X’, on January 15, 2021 at 7:05 p.m.
  3. January 15, 2021 at 4:04 p.m.
  4. January 15, 2021 at 7:05 p.m.

Answer: (a)

79. ‘X’, who is in Agra, makes an offer for sale of second-hand luxury car to ‘Y’, who is Jammu, via an e-mail sent on January 15, 2021 at 2:03 pm. However, the e-mail did not reach ‘Y’ due to some technical error at the server which is located in Delhi. Thereafter, ‘X’ makes a mobile phone call to ‘Y’ on January 15, 2021 at 4:04 pm and makes him the same offer as was made in the e-mail. In the same mobile phone call, the offer is absolutely and unconditionally accepted by ‘Y’ at 4:10 pm. In the given situation, where is the contract concluded?

  1. Delhi
  2. Jammu
  3. Agra
  4. Neither Delhi, Jammu nor Agra as it is an electronic contract.

Answer: (c)

80. “When the words of acceptance are spoken into the telephone, they are put into the course of transmission to the offerer so as to be beyond the power of the acceptor. The acceptor cannot recall them.” In light of the given proposition, which of the following statements is/are true?

I. The communication being instantaneous, the contract immediately arises. II. The communication being instantaneous, the communication of acceptance is immediately complete as against the proposer as well as the acceptor. III. The communication being non-instantaneous, the communication of acceptance is complete as against the acceptor when the words of acceptance are spoken into the telephone. IV. The communication being non-instantaneous, the communication of acceptance is complete as against the proposer when the words of acceptance are spoken into the telephone.

  1. Only I
  2. I and II
  3. III and IV
  4. Only II

Answer: (b)

It is a well settled principle of contract law that parties cannot by contract exclude the jurisdiction of all courts. Such a contract would constitute an agreement in restraint of legal proceedings and contravene Section 28 of the Indian Contract Act, 1872. However, where parties to a contract confer jurisdiction on one amongst multiple courts having proper jurisdiction, to the exclusion of all other courts, the parties cannot be said to have ousted the jurisdiction of all courts. Such a contract is valid and will bind the parties to a civil action. Section 28. Agreements in restraint of legal proceedings, void-Every agreement, – (a) by which any party thereto is restricted absolutely from enforcing his rights under or in respect of any contract, by the usual legal proceedings in the ordinary tribunals, or which limits the time within which he may thus enforce his rights; or (b) which extinguishes the rights of any party thereto, or discharges any party thereto, from any liability, under or in respect of any contract on the expiry of a specified period so as to restrict any party from enforcing his rights, is void to the extent. Parties cannot by agreement confer jurisdiction on a court which lacks the jurisdiction to adjudicate. But where several courts would have jurisdiction to try the subject matter of the dispute, they can stipulate that a suit be brought exclusively before one of the several courts, to the exclusion of the others.

81. ‘A’, a resident of Mumbai, and ‘B’, a resident of Delhi, enter into an agreement for sale and supply of goods. The transaction takes place partly in Mumbai and partly in Delhi. There is a clause in the agreement which stipulates that in the event of a dispute between ‘A’ and ‘B’, the courts in Kolkata would have exclusive jurisdiction to decide the dispute. ‘A’ and ‘B’ agreed to the said clause in order to avoid dispute over choice between the two proper places of jurisdiction- Mumbai and Delhi. In the given situation, which of the following statements is true?

  1. The clause relating to jurisdiction is in restraint of legal proceedings
  2. The clause relating to jurisdiction is not in restraint of legal proceedings
  3. The clause relating to jurisdiction is valid as ‘A’ and ‘B’ have mutually agreed to the same
  4. The clause relating to jurisdiction is valid as its object is lawful.

Answer: (a)

82. ‘A’, a resident of Chennai, and ‘B’, a resident of Bengaluru, enter into an agreement for sale and supply of goods. The transaction takes place partly in Chennai and partly in Bengaluru. There is a clause in the agreement which stipulates that in the event of a dispute between ‘A’ and ‘B’, the courts in Chennai would have exclusive jurisdiction to decide the dispute. ‘A’ and ‘B’ agreed to the said clause in order to avoid dispute over choice between the two proper places of jurisdiction- Chennai and Bengaluru. In the given situation, which of the following statements is true?

  1. The clause relating to jurisdiction is in restraint of legal proceedings
  2. The clause relating to jurisdiction is void
  3. The clause relating to jurisdiction is valid as ‘A’ and ‘B’ have mutually agreed to the same
  4. The clause relating to jurisdiction is valid as courts in Chennai have jurisdiction to decide the dispute.

Answer: (d)

83. ‘A’, a resident of Agra, and ‘B’, a resident of Bhubaneswar, enter into an agreement for sale and supply of goods. The transaction takes place partly in Agra and partly in Bhubaneswar. There is a clause in the agreement which stipulates that in the event of a dispute between ‘A’ and ‘B’, neither of them can approach the court of law or take recourse to any alternative dispute resolution mechanism to settle the dispute. In the given situation, which of the following statements is true?

  1. The clause relating to jurisdiction is not valid as it is in restraint of legal proceedings
  2. The clause relating to jurisdiction is not valid as the clause is vague and ambiguous
  3. The clause relating to jurisdiction is valid as they have not restricted the choice of either party regarding choice of jurisdiction
  4. The clause relating to jurisdiction is valid as no court’s has been ousted by the clause.

Answer: (a)

84. ‘A’, a resident of Ahmedabad, and ‘B’, a resident of Ranchi, enter into an agreement for sale and supply of goods. The transaction takes place partly in Ahmedabad and partly in Ranchi. Clause 6 of the agreement stipulates that in the event of a dispute arising between ‘A’ and ‘B’ within six months of the entering into contract, they can approach a court in either Ahmedabad or Ranchi (as both are proper places of jurisdiction), or take recourse to any alternative dispute resolution mechanism to settle the dispute. Clause 7 of the agreement stipulates that in the event of a dispute arising between ‘A’ and ‘B’ after the expiry of six months of entering into contract, the courts in Chennai would have exclusive jurisdiction to decide the dispute. In the given situation, which of the following statements is true?

  1. Clause 6 is void and Clause 7 is valid
  2. Clause 6 is valid and Clause 7 is void
  3. Both Clause 6 and Clause 7 are valid
  4. Both Clause 6 and Clause 7 are void.

Answer: (d)

85. According to the given passage, which of the following statements is true?

  1. Parties cannot by contract make a choice of jurisdiction
  2. Parties cannot by contract exclude the jurisdiction of all courts
  3. Parties can by contract confer jurisdiction on any court
  4. Parties can by contract extinguish their rights under any contract.

Answer: (b)

The doctrine of res judicata requires that a party should not be allowed to file same matter repeatedly against the other party either in the same court or in other competent court and that the decision given by one court should be accepted as final subject to any appeal, revision or review. The doctrine is founded on the principle that it is in the interest of the public at large that a finality should be attached to the binding decisions pronounced by courts of competent jurisdiction, and it is also in the public interest that individuals should not be vexed twice over with the same kind of litigation. This apart, the object of the doctrine is to ensure that ultimately there should be an end to litigation. Doctrine of res judicata is embodied in Section 11 of the Code of Civil Procedure, 1908 which governs the procedure to be followed in civil matters. Section 11 is inapplicable to writ jurisdictions. The Supreme Court has observed that though the rule is technical in nature yet the general doctrine of res judicata is based on public policy and therefore, it cannot be treated as irrelevant or inadmissible even in dealing with fundamental rights in petitions filed under Article 32 of the Constitution of India. The court observed that if a writ petition filed by a party under Article 226 of the Constitution of India is considered on merits as a contested matter and is dismissed, the decision thus pronounced would continue to bind the parties unless it is otherwise modified or reversed in appeal or other appropriate proceedings permissible under the Constitution of India. It would not be open to a party to ignore the judgment of the High Court and move Supreme Court under Article 32 by an original petition made on the same facts and for obtaining the same or similar orders or writs. If the petition filed in the High Court under Article 226 is dismissed but not on the merits, then the dismissal of the writ petition would not constitute a bar to a subsequent petition under Article 32, however if the petition is dismissed without passing a speaking order, then such dismissal cannot be treated as creating a bar of res judicata.

86. Which of the following is res judicata applicable to?

  1. Civil suits
  2. Writ petitions
  3. None of the above
  4. Both (a) and (b).

Answer: (d)

87. Doctrine of res judicata is applicable to writs if:

  1. The decision is on merits
  2. Order is a speaking order
  3. Both (a) and (b)
  4. Neither (a) nor (b).

Answer: (c)

88. On which of the following is the doctrine of res judicata based?

  1. No one should be vexed twice for the same cause more than once
  2. It is in the interest of public that finality should be attached to the decisions of courts
  3. There should be an end to litigation
  4. All of these.

Answer: (d)

89. Which of the following is correct?

  1. If a writ petition is filed under Article 226 of the Constitution of India and the same is rejected on merits by a speaking order, another petition under Article 32 of the Constitution of India is not maintainable being barred by res judicata
  2. The doctrine of res judicata is founded on the principle that it is in the interest of the public at large that a finality should be attached to the binding decisions pronounced by courts of competent jurisdiction
  3. Technical rule of res judicata only prevents multiple filing of petitions under Article 226 of the Constitution of India between same parties over the same matter and is not applicable to petition under Article 32 of the Constitution of India.
  4. Both (a) and (b).

Answer: (d)

90. Mr. X was dismissed from service by his employer after a proper enquiry. Mr. X challenged his dismissal in High Court by a petition under Article 226 of the Constitution of India. However, the High Court dismissed the petition citing that Mr. X has an alternative remedy available. Mr. X took recourse to the alternative remedy before the appropriate forum, but Mr. X’s legal action is opposed by the employer on the basis of res judicata. Based on these facts, which of the following is the most appropriate?

  1. Res judicata is applicable
  2. Res judicata is not applicable
  3. Since the matter relates to livelihood and life of the person, technical rule of res judicata should not be applied
  4. Both (b) and (c).

Answer: (b)

Compassionate appointment is an exception to the general rule of appointment which is a way of providing employment to the family of the deceased employee on compassionate grounds. The objective is only to provide solace and succour to the family in difficult times and, thus, its relevancy is at that stage of time when the employee passes away. The mere death of an employee in harness does not entitle his family to such source of livelihood. The authority concerned has to examine the financial condition of the family of the deceased, and it is only if it is satisfied that, but for the provision of employment, the family will not be able to meet the crisis that the job is offered to the eligible member of the family. It was further asseverated in the said judgment that compassionate employment cannot be granted after a lapse of reasonable period as the consideration of such employment is not a vested right which can be exercised at any time in the future. It was further held that the object of compassionate appointment is to enable the family to get over the financial crisis that it faces at the time of the death of sole breadwinner. Thus, compassionate appointment cannot be claimed or offered after a significant lapse of time and after the crisis is over.

91. Which of the following correctly states the intent behind the application of compassionate appointment?

  1. The norms, at the time of death of the government employee will be applicable and not the norms at the time of making an application
  2. A dependent of a government employee cannot demand consideration of his/her application
  3. The norms of the governmental or public authorities are not considered while applying for compassionate appointment
  4. The norms prevailing on the date of consideration of the application should be the basis for consideration of claim for compassionate appointment.

Answer: (d)

92. Which of the following is not correct about the rule of compassionate appointment?

  1. Compassionate appointment is given to the family members of the deceased irrespective of their financial status
  2. It is to mitigate the hardship caused to the family members after the death of earning member of the family
  3. Compassionate appointment cannot be granted as a matter of vested right
  4. Compassionate appointment cannot be made in the absence of rules and regulations issued by the government or a public authority.

Answer: (a)

93. Mr. Y, son of Mr. X, made a representation before ABC government company on January 4, 2018 that he should be given appointment on compassionate grounds as his father died during his employment in the company in 2000. Consider the given facts and decide whether Mr. Y is entitled to get compassionate appointment.

  1. Y is entitled to get compassionate appointment
  2. Y is not entitled for compassionate appointment as a long period has elapsed since the death of his father
  3. Y is entitled to get compassionate appointment depending on policy of the company in which his father was working
  4. Y is not entitled to get compassionate appointment as it is the discretion of the company to refuse the said appointment.

Answer: (b)

94. The Government of ‘N’ formulated a scheme for providing compassionate appointment to the dependants of government servants who retired on medical invalidation. By a further notification, the benefit of the scheme was restricted to cases where the government servants retired on medical invalidation, at least five years before attaining the age of superannuation. Consider the given facts, and decide which of the following is correct in relation to the validity of this rule of compassionate appointment?

  1. The scheme is valid subject to the approval of the dependents
  2. The scheme is not valid as the offer of compassionate appointment to the dependent of a government servant who is medically invalidated is not an exception to the general rule
  3. The scheme is not valid as it is unconstitutional
  4. The scheme is valid as it is not arbitrary and the government has the right to formulate such rules.

Answer: (d)

95. Which of the following is not correct regarding the nature of appointment on compassionate grounds?

  1. A request for compassionate appointment by the dependent relatives of the deceased must be preferred without any undue delay
  2. The general rule of appointment may not be always applicable to compassionate appointments
  3. The immediacy of the need is not the basis for the state to allow the benefit of compassionate appointment
  4. It is a benefit given to the family members at the time of distress.

Answer: (c)

Marriage is necessarily the basis of social organisation and the foundation of important legal rights and obligations. The importance and imperative character of the institution of marriage needs no comment. In Hindu law, marriage is treated as a Samskara or a sacrament. The Hindu Marriage Act, 1955 introduced monogamy as a law of marriage among Hindus by virtue of Section 5 clause (i) which is essentially the voluntary union for life of one man with one woman to the exclusion of all others. It enacts, “neither party must have a spouse living at the time of marriage”. The expression ‘spouse’ here used, means a lawfully married husband or wife. Before a valid marriage can be solemnised, both parties to such marriage must be either single or divorced or a widow or a widower and only then they are competent to enter into a valid marriage. If at the time of performance of the marriage rites and ceremonies, one or other of the parties had a spouse living and the earlier marriage had not already been set aside, the later marriage is no marriage at all. The Supreme Court in Bhaurao Shankar Lokhande v. State of Maharashtra, [AIR 1965 SC 1564] held, “Prima facie, the expression ‘whoever marries’ in Section 494 of the Indian Penal Code, 1860 (which defines the offence of bigamy) must mean ‘whoever marries validly’ or ‘whoever marries and whose marriage is a valid one’. If marriage is not valid according to the law applicable to the parties, no question arises of its being void by reason of its taking place during the life of the husband or wife of the person marrying. One of the conditions of a valid marriage under the Hindu Marriage Act, 1955 is that it must be ‘solemnised’. Further, Section 13 (2) of the Act provides for grounds of divorce to wife and states, “A wife may also present a petition for the dissolution of her marriage by a decree of divorce on the ground that in the case of any marriage solemnized before the commencement of this Act, that the husband had married again before such commencement or that any other wife of the husband married before such commencement was alive at the time of the solemnization of the marriage of the petitioner: Provided that in either case the other wife is alive at the time of the presentation of the petition”.

96. Mr. A, a 40-year old male Hindu, was married to Ms. B, a 36-year old female Hindu. Mr. A fell in love with his colleague- Ms. C, a 22-year old female Christian. On April 8, 2020, Mr. A declared Ms. C as his wife in front of all his colleagues, family members and relatives. Based on the given facts, decide the liability of Mr. A as per Hindu law.

  1. A is not liable for the offence of bigamy as Ms. C is a Christian
  2. A is liable for the offence of bigamy as mere declaration also amounts to solemnisation
  3. A is not liable for the offence of bigamy as he has not solemnised his marriage with Ms. C
  4. A is liable for the offence of bigamy as there was no intention to marry.

Answer: (c)

97. Which of the following statements correctly expresses the interpretation of the word ‘solemnise’ under Hindu law?

  1. Solemnisation means celebrating the marriage with proper customary rites and ceremonies of either party to a marriage
  2. Solemnisation includes and means promising each other a lifetime of happiness
  3. Solemnisation is not necessary in modern marriages when registration of marriage is complete
  4. Solemnisation is a mere formal practice and not a mandate.

Answer: (a)

98. Which of the following is not correct regarding the law of monogamy among Hindus?

  1. Monogamy is a union of a man and woman which provides their relation a social and legal recognition
  2. Monogamy as a law was abolished after 1955
  3. The second marriage during the subsistence of a first valid marriage is void
  4. If a husband solemnises a second marriage during the subsistence of first marriage, it is not an offence, however, it is an offence if a wife commits the same act.

Answer: (b)

99. On the basis of Section 13 of the Hindu Marriage Act, 1955, which of the following statements does not relate to the remedy?

  1. The object of the Section was to provide an opportunity to the wife in the form of remedy of divorce
  2. The introduction of this ground of divorce is unnecessary as it disturbs the sanctity of marital institution
  3. A remedy is only available under this Section if the husband has the other wife living
  4. The living status of either spouse is immaterial to claim the remedy.

Answer: (b)

100. Mr. P, a 28-year-old male Hindu was legally married to Ms. Q, a 26-year-old female Hindu. Mr. P converts to Islam to marry Ms. N, a 30-year-old Sunni female Muslim. Consider the statement and decide whether Ms. Q has the remedy to file a complaint for the offence of bigamy against Mr. P?

  1. Yes, Ms. Q can file a complaint subject to the approval by Ms. N
  2. No, Ms. Q cannot file a complaint as it defeats the very purpose of her marriage with Mr. P
  3. No, Ms. Q cannot file a complaint as Mr. P converted to another religion and the offence of bigamy will only be attracted when the parties are Hindus
  4. Yes, Ms. Q has the remedy of filing a complaint for the offence of bigamy.

Answer: (d)

The Indian Penal Code, 1860 does not define ‘consent’ in positive terms, but what cannot be regarded as ‘consent’ under the Code is explained by Section 90. Section 90 reads as follows: “90. Consent known to be given under fear or misconception – A consent is not such a consent as is intended by any section of this Code, if the consent is given by a person under fear of injury, or under a misconception of fact, and if the person doing the act knows, or has reason to believe, that the consent was given in consequence of such fear or misconception;…” Consent given firstly under fear of injury and secondly under a misconception of fact is not ‘consent’ at all. That is what is enjoined by the first part of Section 90. These two grounds specified in Section 90 are analogous to coercion and mistake of fact which are the familiar grounds that can vitiate a transaction under the jurisprudence of our country as well as other countries. The factors set out in the first part of Section 90 are from the point of view of the victim. The second part of Section 90 enacts the corresponding provision from the point of view of the accused. It envisages that the accused too has knowledge or has reason to believe that the consent was given by the victim in consequence of fear of injury or misconception of fact. Thus, the second part lays emphasis on the knowledge or reasonable belief of the person who obtains the tainted consent. The requirements of both the parts should be cumulatively satisfied. In other words, the court has to see whether the person giving the consent had given it under fear of injury or misconception of fact and the court should also be satisfied that the person doing the act i.e. the alleged offender, is conscious of the fact or should have reason to think that but for the fear or misconception, the consent would not have been given. This is the scheme of Section 90 which is couched in negative terminology. Section 90 cannot, however, be construed as an exhaustive definition of consent for the purposes of the Indian Penal Code, 1860. The normal connotation and concept of ‘consent’ is not intended to be excluded. Various decisions of the High Court and of Supreme Court have not merely gone by the language of Section 90, but travelled a wider field, guided by the etymology of the word ‘consent’.

101. ‘A’, a man, promises ‘B’, a woman that he will marry her if she has sexual intercourse with him. ‘B’ agrees, but after having sexual intercourse, ‘A’ flees and never contacts ‘B’ again. In the given situation, which statement is true?

  1. Consent for sexual intercourse was given under misconception of fact
  2. Consent for sexual intercourse was given under fear of injury
  3. Consent for sexual intercourse was given under undue influence
  4. Consent for sexual intercourse was given without any misconception or fear of injury.

Answer: (a)

102. According to the given passage, a person is said to ‘consent’ to the doing of an act if:

  1. A person agrees to do an act without misconception of fact or fear of injury
  2. The person to whom agreement is signified has no reason to believe that the agreement is being given under misconception of fact or fear of injury
  3. A person agrees to do an act without misconception of fact or fear of injury and the person to whom agreement is signified has no reason to believe that the agreement is being given under misconception of fact or fear of injury.
  4. A person agrees to do an act without having regard to the consequences of fear of injury or misconception of fact.

Answer: (c)

103. According to your understanding of the given passage, why is negative terminology used to explain the meaning of consent under Section 90?

  1. To emphasise on the factors that vitiate consent
  2. To emphasise on the point of view of victim or the person who gives consent
  3. To emphasise on the point of view of the person who receives consent
  4. To emphasise on the non-exhaustive scope of definition of consent.

Answer: (d)

104. ‘X’, a man, promises ‘Y’, a woman that he will marry her if she has sexual intercourse with him. ‘Y’ agrees and they have sexual intercourse. Thereafter, ‘X’ assures ‘Y’ that they will get married, but X’s family is opposed to the marriage even after X’s attempts to convince them. Therefore, ‘X’ refuses to marry ‘Y’. In the given situation, which statement is true?

  1. Consent for sexual intercourse was received knowing that the consent is given under misconception of fact
  2. Consent for sexual intercourse was given without any misconception of fact or fear of injury
  3. Consent for sexual intercourse was received knowing that the consent is given under fear of injury
  4. Consent for sexual intercourse was given under misconception of fact and received misconception of fact.

Answer: (b)

105. According to the given passage, which of the following statements is not true?

  1. Misconception of fact vitiates consent
  2. Fear of injury vitiates consent
  3. Misconception of fact obviates the necessity of consent
  4. Fear of injury is analogous to coercion.

Answer: (c)

Logical Reasoning The COVID-19 pandemic and the unmatched mental health challenges have made it more crucial than ever that we continue to make strides towards understanding the concept of mental health stigma and how we might tackle it around the world. Graham Thornicroft, a practising psychiatrist, who is extensively and deeply involved in mental health stigma research at the Institute of Psychiatry, Psychology and Neurosciences at King’s College London has divided stigma into three components-knowledge, attitude and behaviour. The last behaviour emerges from social isolation, such as what we are experiencing during the pandemic, as well as exclusion from mainstream activities and citizenship. In higher-income countries, stigma rates may be greater than other countries, perhaps because of the pressure to excel. In low-income countries, one can be unwell and still play an active social and productive role somewhere as there are many such roles to play within the family and in society. Enhancing contact with people who have experienced mental health problems is the best way to reduce stigma. To date, most people with mental illness remain silent about their condition, avoiding discussing their problems for fear of losing face, damaging their reputation or jeopardising their family status. Having a space where they may be welcomed and listened to, rather than judged, will go a long way towards enabling them to share their experiences. In a small part of rural Andhra Pradesh, researchers used posters, pictures, drums, and a short street play, as an intervention technique to reduce mental health stigma. An actor portrayed a person’s journey through mental health crises and setbacks before receiving support and showing hope, improvement and recovery. People assembled around the stage, willing to talk about and discuss what they saw, even two to three years after the event.

106. What is the central idea in the passage as conveyed by the author?

  1. Mental health is not an obstacle for people seeking care
  2. Mental health is a domestic phenomenon
  3. In low-income countries, mental health does not receive the adequate attention and treatment
  4. Mental health issues have only arisen after the COVID-19 pandemic globally.

Answer: (c)

107. As per the passage, which of the following approaches can be most effective to curb the issue of mental health?

  1. Mental health can only be curbed when people will stop sharing their emotions openly
  2. To develop a sense of acceptance among people suffering from mental health and let them heal gradually
  3. Experiences shared and discussed on a larger level is a major hindrance for better mental health conditions globally
  4. A person should not express about his/her failures which enhances the mental health problems.

Answer: (b)

108. Which of the following notions is expressed in the passage to enunciate the significance of issue of mental health?

  1. The issue of mental health has received the required recognition and deliberation during the COVID-19 pandemic
  2. Mental health will be reduced automatically after the COVID-19 pandemic
  3. Mental health improves when social isolation increases
  4. Mental health is an illusionary notion developed during COVID-19 pandemic to strengthen the pharmaceutical industries.

Answer: (a)

109. “In low-income countries, one can be unwell and still play an active social and productive role somewhere as there are many such roles to play within the family and in society”. Which of the following conclusions can be drawn from the above statement?

  1. Stigma rates in lower-income countries are higher as compared to higher income countries
  2. Stigma rates in higher-income countries may be more due to a pressure of performing and to excel
  3. Mental stigma is not related to the economic conditions of a country
  4. Higher-income countries require people to perform multiple roles at their homes.

Answer: (b)

110. Which of the following weakens the author’s idea of ‘having a space where they may be welcomed and listened to, rather than judged’?

  1. People who tend to share their emotions tend to have mental peace and happiness
  2. Societal role is massive in embracing people suffering from mental health ailments
  3. People sharing their ideas and expressions publicly is a practice which needs to be promoted
  4. It is a human tendency to form judgments about other people and this being a healthy practice should be encouraged at large scale.

Answer: (d)

One of the most important challenges for Indian diplomacy in the subcontinent is to persuade its neighbours that India is an opportunity, not a threat. Far from feeling in any way besieged by India, they should be able to see it as offering access to a vast market and to a dynamic, growing economy which would provide their own economies with far greater opportunities than more distant partners (or even their own domestic markets) could provide. This would go beyond economic benefits: as David Malone argues, “Economic cooperation represents the easiest ‘sell’ to various constituencies within the countries of the region. Were this to prove successful, cooperation on more divisive and sensitive issues, such as terrorism, separatism, insurgency, religious fundamentalism, and ethnic strife, could be attempted with greater chances of success.” Winds of change are blowing in South Asia. There is a definite consolidation of democracy in all the countries of the region, every one of which has held elections within the last three years. Some of our neighbours have made significant strides in surmounting internal conflict and others are in the process of doing so. If India has to fulfil its potential in the world, we have no choice but to live in peace with our neighbours, in mutual security, harmony and cooperation. Just as Nehru left Robert Frost’s immortal lines “Miles to go before I sleep” on his bedside table when he died, Shastri kept some lines of the founder of the Sikh faith, Guru Nanak, on his desk. When translated into English they read: “O Nanak! Be tiny like the grass, for other plants will wither away, but grass will remain ever green.” Shastri was seen by many Indians of exalted ambition as a tiny man, but he had the mind and heart of a giant. His vision of peaceful coexistence with our neighbours, through adopting the demeanour, the modesty and the freshness of grass, may well be the best way for India to ensure that its dreams remain evergreen in its own backyard.

111. As per the passage, which of the following is a challenge for India?

  1. To develop an intimidating surrounding for its neighbours
  2. To manifest India as a land of opportunities
  3. To restrict access to economic activities to its neighbours
  4. To propagate the practice of threatening neighbours for greater opportunities.

Answer: (b)

112. “If India has to fulfil its potential in the world, we have no choice but to live in peace with our neighbours, in mutual security, harmony and cooperation.” In the context of the statement, which of the following strengthens the author’s opinion?

  1. Peace and prosperity cannot work in consonance
  2. India should assert its might over its neighbours
  3. Peaceful coexistence with neighbours is the key to India’s stellar growth
  4. India can offer myriad opportunities for its neighbours by working in seclusion.

Answer: (c)

113. Which of the following is the central theme of the above passage?

  1. The objective of attaining peaceful co-existence and cooperation with the neighbours
  2. The dominance of India on its neighbouring countries
  3. Emergence of India as a global power
  4. Prioritising the economic opportunities for India.

Answer: (a)

114. Which of the following is correct expression of the author’s opinion as stated in passage?

  1. India should strive for greater economic integration which should be politically and administratively feasible
  2. India can see borders as barriers, and border areas as buffer zones but not as gateways of opportunity
  3. India’s stellar economic growth is thwarting her own opportunities
  4. It is not in India’s interest to be generous to neighbouring countries.

Answer: (a)

115. Which of the following can be inferred from the above passage?

  1. There is no nexus between cooperative existence and economic growth of a nation
  2. Unless India’s region becomes more self-centred, India is unlikely to develop into more than a regional power
  3. India can only become a great power if it works in isolation to achieve the desired goals
  4. India cannot aspire to be a great power without the cooperation of neighbours.

Answer: (d)

The critique of school as an institution has developed and grown in the past half a century. Education theorist Everett Reimer wrote School is Dead in the 1960s. Most schools are caged jails, where an alien curriculum designed by some ‘experts’ is thrust down a child’s gullet. Today, many schools are gargantuan corporate enterprises with thousands of children on their rolls, and for all practical purposes they are run like factories, or better still like mini-armies. The website of a private school in Lucknow boasts of 56,000 students, for instance. But progressive thinkers have always envisioned ‘free schools’ for children. The great Russian novelist, Leo Tolstoy, founded a school for the children of poor peasants at his home, Yasnaya Polyana, without any strict schedule, homework or physical punishment. Maria Montessori was the first Italian woman to become a doctor; she went on to work out the ‘stages of development’ in children which became the basis for her educational philosophy, which too emphasised children’s freedom and choice. Tagore’s critique of rote learning is articulated in the classic tale ‘The Parrot’s Training’ (Totaakahini). Perhaps, the longest lasting libertarian school in the world is Summerhill. It was founded in 1921, a hundred years ago in England, by A.S. Neill with the belief that school should be made to fit the child rather than the other way round. The 1966 Kothari Education Commission’s recommendation for a common school system was never implemented. Today, which school a child goes to depends on her socioeconomic status. The pandemic has furthered and exacerbated this divide. COVID-19 hit parents economically. The digital divide between the rich and poor has also widened. The poor do not have access to mobiles, laptops and internet connectivity. In such a scenario, one can try and conceive of neighbourhood learning spaces.

116. What has the author conveyed regarding the school system in India?

  1. The school system in India is an advanced system which focuses on developing the personality of students
  2. The schools in India are segregated based on socio-economic status
  3. The schools staunchly promote the interest of every student enrolled to ensure quality education
  4. The pandemic has aided the students a greater access to the technology and learning spaces.

Answer: (b)

117. Which of the following can be deduced from the above passage?

  1. Social segregation must be eliminated for furtherance of education in schools
  2. The COVID-19 pandemic has lubricated the teaching-learning process in schools
  3. Schools should be divided on the basis of the financial capabilities only
  4. Private schools should be encouraged to enroll economically poor students irrespective of their caste or religion.

Answer: (a)

118. As per the above passage, which of the following does not correctly represent the author’s view regarding imparting of education during the pandemic?

  1. There has been a disruption of education systems due to the pandemic
  2. The inclusion of technology has widened the canvas of distance learning education for all, albeit its access is the biggest concern
  3. The pandemic has unearthed and stimulated innovative approaches to learning
  4. The crisis is worsened by reduction of opportunities for many children.

Answer: (c)

119. Which of the following can be utilised to develop a healthy environment for students’ learning in schools?

  1. Individualised teaching should not be followed for growth of students
  2. Students should only be permitted to opt for only few subjects as elective courses
  3. Community learning should be encouraged in schools
  4. A fixed and robust curriculum is the need of the hour.

Answer: (c)

120. In the above passage, which of the following cannot be referred as a characteristic of a ‘free school’?

  1. It is an alternative to mainstream and conventional form of schooling
  2. Free schools provide a democratic platform to the students to voice their opinions
  3. The courses offered at free schools are optional for students depending on their interests
  4. Specific choice of subjects and a fixed schedule are quintessential features of a free school.

Answer: (d)

Asia is at the front line of climate change. Extreme heat in India, Pakistan, and Bangladesh, wildfires in Australia, typhoons in Japan, are real and present dangers and likely to become more frequent as climate change intensifies. McKinsey’s report on ‘Climate risk and response in Asia’, finds that, without adaptation and mitigation, Asia is expected to experience more severe socioeconomic impacts of climate change than other parts of the world. Large cities in the Indian Subcontinent could be among the first places in the world to experience heat waves that exceed the survivability threshold. Just as information systems and cybersecurity have become integrated into corporate and public-sector decision making, climate change will also need to feature as a major factor in decisions. Climate science tells us that some amount of warming over the next decade is already locked in due to past emissions, and temperatures will continue to rise. India anticipates a significant infrastructure build out over the next decades with projects worth $1.77 trillion across 34 sub sectors, according to the National Infrastructure Pipeline. Robust regulations around outdoor work could significantly reduce the economic risk of lost hours as well as the toll on life from heat waves. The good news is that we have started to see some Indian states and cities pursuing such policies. Ahmedabad City Corporation introduced a heat action plan- the first of its kind in India in response to the 2010 heat wave that killed 300 people in a single day. The city now has a heat-wave early warning system, a citywide programme of roof reflectivity to keep buildings cool, and teams to distribute cool water and rehydration tablets during heat waves. Renewable energy has grown rapidly in India and can contribute 30 per cent of gross electricity generation by 2030, according to the Central Electricity Authority.

121. Which of the following cannot be inferred from the above passage?

  1. It is imperative to lessen our vulnerability to harmful effects of climate change
  2. Increase in mitigation strategies will eventually lead to an increase in pollutants
  3. Reduction of emissions which are heat-trapping and harmful for environment should be our primary goal
  4. The infrastructure development plans need to be climate friendly.

Answer: (b)

122. Consider the following statements: I. The impact of increasing heat is more on the ones who are economically poor and engaged in outdoor employment. II. Risk to the environment is directly related to change in climate. III. A localised understanding of climate risk is essential to mitigate the risk. As per the above passage, which of these statements is implicit?

  1. I only
  2. I and II
  3. I, II and III
  4. I and III.

Answer: (c)

123. Which of the following strategies is not appropriate to reduce the climate change risk?

  1. Decreasing energy efficiency
  2. Carbon Pricing System
  3. Decarbonising industrial operations and buildings
  4. Investments in adaptive technologies.

Answer: (a)

124. As per the above passage, which of the following is a major challenge to control the climate change?

  1. Reliance on non-renewable energy is significant for economic growth
  2. Renewable energy is now utilised in many sectors to curb the increasing risk of climate change
  3. Remedial actions for climate change are encouraged to lessen the risk of global warming
  4. Regulated industrialisation keeps a balance between growth and environment.

Answer: (a)

125. Consider the following statement- “Just as information systems and cybersecurity have become integrated into corporate and public-sector decision making, climate change will also need to feature as a major factor in decisions.” Which of the following is the most appropriate explanation for the above statement?

  1. Decision making by organisations should not depend on the climate conditions which may lead to hampering their growth
  2. Monetary investments in adoption of climate friendly tools is not an urgent need
  3. The understanding of climate risk should not be integrated into decision making
  4. Organisations must incorporate climate risk, build the necessary infrastructural capabilities and tools to diagnose risks.

Answer: (d)

COVID-19 infections are once again on the rise with daily infections crossing 60,000 per day last week. This is considerably higher compared to the reported infections during the same period last year when the numbers were less than 500 per day. What is obvious is that the pandemic is far from over despite the availability of vaccines. However, unlike last year, the response this time has been muted with no nationwide lockdown. One of the reasons for the differing responses is the lesson from the unintended consequences on the economy of the strict lockdown last year. While aggregate estimates on the growth rate of GDP showed a sharp contraction in economic activity (the economy shrunk by 24 per cent in the April-June quarter of 2020) the impact on lives and livelihoods is still unfolding even though the sharp contractionary phase seems behind us. The extent of the loss of lives and livelihoods is becoming clear only now, with detailed data from the Periodic Labour Force Surveys (PLFS) – the latest round of which is for the April-June quarter of 2020. This is the first official report on the estimates for the quarter, which witnessed the worst impact with the lockdown in force until the middle of May. Visuals of thousands of migrants walking back to their villages are still fresh in the mind. While many have returned to urban areas in the absence of jobs in rural areas, many did not. The PLFS, which captures the employment-unemployment situation in urban areas, provides some clues to what happened. The estimates from PLFS are broadly in line with estimates available from other privately conducted surveys, notably the unemployment surveys of the Centre for Monitoring Indian Economy (CMIE). According to the PLFS April-June 2020 round, the urban unemployment rate for the population above the age of 15 was 20.8 per cent, which is close to the monthly average for the same quarter from CMIE at 19.9 per cent. The CMIE data, however, does suggest a sharp decline in June compared to April and May. Similar to the CMIE data, the PLFS data also shows a sharp rise in the unemployment rate which more than doubled compared to the unemployment rate in the preceding quarter of January-March 2020 at 9.1 per cent and 8.8 per cent in the same quarter (April-June) of 2019. While one in five persons above the age of 15 was unemployed during April-June 2020, the unemployment rate among the 15-29-year-olds was 34.7 per cent – every third person in the 15-29 age group was unemployed during the same period. These are staggering numbers, but not surprising. While the lockdown certainly contributed to the worsening of the employment situation, particularly in urban areas, the fact that the economy was already going through severe distress as far as jobs are concerned is no longer surprising. Between 2016-17 and 2019-20, growth decelerated to 4 per cent, less than half the 8.3 per cent rate in 2016-17. The fact that the economy has not been creating jobs predates the economic shocks of demonetisation and the hasty roll-out of GST. The PLFS data from earlier rounds have already shown the extent of the rise in unemployment compared to the employment unemployment surveys of 2011-12. The unemployment rates in urban areas for all categories increased by almost three times between 2011-12 and 2017-18. On an internationally comparable basis, the unemployment rate among the 15-24-year-olds in 2017-18 was 28.5 percent, which makes the youth unemployment rate in India amongst the highest in the world, excluding small countries and conflict-ridden countries. Since then, it has only worsened or remained at that level.

126. According to the author, which of the following is a plausible explanation for India’s prevalent crisis of unemployment?

  1. The previously existing employment crunch is worsened due to the imposition of nationwide lockdown during the COVID-19 pandemic
  2. Imposition of nationwide lockdown during the COVID-19 pandemic reduced economic activity
  3. Imposition of nationwide lockdown during the COVID-19 pandemic led to migration of workforce from urban to rural areas
  4. There is a sharp decline in urban unemployment rate for the population above the age of 15.

Answer: (a)

127. According to the author, what is current state of Indian economy?

  1. There is a sharp contraction in economic activity
  2. The aftermath of a sharp contraction in economic activity is being experienced
  3. The unemployment rates in urban areas for all categories are increasing by almost three times
  4. Youth unemployment rate in India is at par with that in small and conflict-ridden countries.

Answer: (b)

128. Which of the following statements cannot be inferred from the given passage?

  1. Loss of lives and livelihoods is attributable to the imposition of nationwide lockdown during the COVID-19 pandemic
  2. Decline in economic growth is attributable to the imposition of nationwide lockdown during the COVID-19 pandemic
  3. Rise in unemployment is attributable to the migration of members of workforce from urban to rural areas
  4. Rise in unemployment is attributable to the imposition of nationwide lockdown during the COVID-19 pandemic.

Answer: (c)

129. Which of the following strengthens the author’s argument that the decline in economic growth and rise in unemployment precedes the onset of the COVID-19 pandemic?

  1. While one in five persons above the age of 15 was unemployed during April-June 2020, the unemployment rate among the 15-29-year-olds was 34.7 per cent – every third person in the 15-29 age group was unemployed during the same period
  2. According to the PLFS April-June 2020 round, the urban unemployment rate for the population above the age of 15 was 20.8 per cent, which is close to the monthly average for the same quarter from CMIE at 19.9 per cent
  3. Between 2016-17 and 2019-20, growth decelerated to 4 per cent, less than half the 8.3 per cent rate in 2016-17
  4. The estimates from PLFS are broadly in line with estimates available from other privately conducted surveys, notably the unemployment surveys of the Centre for Monitoring Indian Economy (CMIE).

Answer: (c)

130. According to the given passage, which of the following statements is correct?

  1. Unemployment rates in urban areas for all categories increased by almost three times over the preceding decade
  2. Employment rates in urban areas for all categories increased by almost three times over the preceding decade
  3. Unemployment rates in urban areas for all categories increased by almost three percent over the preceding decade
  4. Employment rates in urban areas for all categories increased by almost three percent over the preceding decade.

Answer: (a)

On the day of writing this, India had reported 116 deaths from COVID-19. In contrast, the US, with around one-fourth the population of India, reported 1,897 deaths, or 16 times the daily deaths as India. The UK, which has one-twentieth the population of India, reported 592 deaths, or 5 times the daily deaths as India. On other metrics too-new cases, active cases-the Indian curve has flattened. If and when the UK and the US achieve what we have, there will be major celebrations. Such low death rates would be seen as a victory of the government, citizens and science over the dreaded coronavirus. However, because we are India, we don’t get as much credit. We are considered poor, third-world and untrustworthy, incapable of achieving something like this on our own. Instead of learning from India’s experience, the first instinct is to doubt Indian data. We aren’t counting the cases right, we aren’t doing enough tests, we don’t classify the deaths properly-the list of doubts goes on and on. This, even as the tests have only increased, positivity rate has dropped and almost all Indian hospitals are seeing a drop in COVID-19 admissions and fatalities. To think that the Deep Indian State is capable of fudging data at the level of every district and every state, and sustaining this façade for months is giving it way too much credit. Conspiracies require enormous co-ordination and effort and it isn’t quite how things work in India. Given that you can check corona data at every ward level, it is also impossible to fudge data, not to mention create a downwards curve that is moving in the same direction in virtually every corner of India. In terms of testing, while a case might be made for a lot of Indians not getting tested, it is also true that random testing has increased in the last few months. Domestic flyers into Maharashtra from many states for instance, have to get a COVID-19 test done irrespective of symptoms. If there was rampant corona, we would see a spike in cases from just these flyers. It may be hard for people to accept this reality but almost all evidence points to the fact India has flattened the corona curve, while the US, UK and most of Europe still haven’t. What is even more remarkable about India’s achievement is that it has managed to do this without draconian lockdowns (apart from the two months in April-May 2020). In fact, cases have dropped even as India opened up more.

131. Which of the following strengthens the author’s argument that India has tackled the COVID-19 pandemic better than most advanced nations?

  1. India’s positivity rate of COVID-19 cases and admissions in Indian hospitals are reduced
  2. India’s counting of the number of COVID-19 cases is incorrect
  3. India’s testing facilities for COVID-19 cases are inadequate
  4. India’s classification of COVID-19 cases is inaccurate.

Answer: (a)

132. Which of the following weakens the author’s argument that India has tackled the COVID19 pandemic better than most advanced nations?

  1. India lacks the co-ordination and effort to execute conspiracies
  2. Many Indians are possibly not getting tested
  3. Evidence points to the fact India has flattened the corona curve
  4. India has not imposed draconian lockdowns.

Answer: (b)

133. “To think that the Deep Indian State is capable of fudging data at the level of every district and every state, and sustaining this façade for months is giving it way too much credit.” In this statement, the author indicates that:

  1. There are unauthorised networks of power operating independently of the Indian state’s political leadership in pursuit of their own agenda and goals
  2. There are authorised networks of power operating independently of the Indian state’s political leadership in pursuit of their own agenda and goals
  3. There may be unauthorised networks of power operating independently of the Indian state’s political leadership in pursuit of their own agenda and goals, but their influence is questionable
  4. There may be unauthorised networks of power operating independently of the Indian state’s political leadership in pursuit of their own agenda and goals, and their influence is deep and pervasive.

Answer: (c)

134. According to the given passage, which of the following statements is not true?

  1. Developed nations are reluctant to learn from India’s experience as Indian data is doubtful
  2. India does not get as much credit for tackling the COVID-19 pandemic
  3. India is seen as a poor, third-world country
  4. India is seen as untrustworthy and incapable of tackling the COVID-19 pandemic independently.

Answer: (a)

135. According to the given passage, which of the following statements is true?

  1. COVID-19 cases in India have reduced because of reduction in restrictions
  2. COVID-19 cases in India have increased with reduction in restrictions
  3. COVID-19 cases in India have reduced with increase in restrictions
  4. COVID-19 cases in India have reduced despite the reduction in restrictions.

Answer: (d)

Quantitative Techniques Two renowned international Software companies, namely Pollaris and Contigent, started their business in the year 2007 and both the companies were in competition with each other in profit making. Pollaris earned 30 per cent profit in the year 2007 and 2008, and further increased it to 40 per cent in 2009. However, its profit percentage decreased to 20 per cent in the year 2010. On the other hand, Contigent opened with 40 per cent profit in 2007, but slowly decreased to 35 per cent in 2008 and 30 per cent in 2009. Interestingly, both the companies increased their profit percentage in the later year considerably. Pollaris increased its profit percentage to 35 per cent in 2011 and 50 per cent in 2012; simultaneously, Contigent increased its profit percentage to 45 per cent in 2010, 50 per cent in 2011 and reached 60 per cent in the year 2012. As there is a need to understand the income and expenditure for the better performance of both companies in the future, answer the following questions.

136. What is the increase in profit of Contigent Company in percentage from year 2011 to 2012?

  1. 10%
  2. 20%
  3. 15%
  4. Cannot be determined.

Answer: (b)

137. What percentage of the total profit making of Pollaris Company in 2011 and 2012 is the total profit making of Contigent Company in 2007 and 2008?

  1. 3%
  2. 3%
  3. 3%
  4. 3%.

Answer: (a)

138. If the income of Contigent Company in 2008 was 200 crores, what was its profit in 2009?

  1. ₹5 Crore
  2. ₹15 Crore
  3. ₹ 153 Crore
  4. Cannot be determined.

Answer: (d)

139. What is the difference between the company with highest annual average profit percentage and that of the company with lowest annual average profit percentage?

  1. 17
  2. 33
  3. 17
  4. 33.

Answer: (c)

140. What is the percentage increase in profit of Pollaris Company from year 2010 to 2011?

  1. 86
  2. 75
  3. 175
  4. Cannot be determined.

Answer: (b)

A newly formed state government wants to bring more development in the state. Therefore, the government proposed to launch various welfare programmes. Before bringing up any welfare programme, the state government intended to understand the population percentage of the state by age groups, so that the government could plan the welfare programmes accordingly. The state government found that the state’s 30 per cent of the population were children between the age group of 0-15. Next to child population, 17.75 per cent of the population were adolescents between the age group of 16 and 25. The early adult population, i.e., the age groups 26 to 35 were 17.25 per cent, 36 to 45 were 14.50 per cent, respectively. The population who are between the age group of 46 to 55 constitute 14.25 per cent and the elderly population of the state, i.e., 56 to 65 (5.12%) and 66 above (1.13%) was comparatively less than the other age groups. To get a better clarity, the state government concerned is seeking the answers to following questions:

141. Out of every 5,600 persons, what is the number of persons below the age of 26 years?

  1. 2515
  2. 1746
  3. 1660
  4. 2674.

Answer: (d)

142. There are 400 million people below 36 years. How many million (approx.) people are in the age group 56-65?

  1. 72 million
  2. 75 million
  3. 50 million
  4. 30 million.

Answer: (c)

143. If there are 20 million people in the age group 56 to 65, what is the difference between the number of people in the age groups 16-25 and 46-55?

  1. 6 million
  2. 18 million
  3. 4 million
  4. 7 million.

Answer: (b)

144. If the difference between the number of people in the age groups 46-55 and 26-35 is 15.75 million, then total population of the state is approximately?

  1. 23 million
  2. 390 million
  3. 400 million
  4. 525 million.

Answer: (d)

145. Which age group accounts for the maximum population in the state?

  1. 16 to 25
  2. 26 to 35
  3. 36 to 45
  4. None of the above.

Answer: (a)

COVID-19 pandemic turned life upside-down for many families across the world. Mr.Abhishek was working as a sale executive in a famous Marketing Company. Due to COVID19 pandemic lockdown and restrictions, Mr. Abhishek was not able to achieve his monthly targets which resulted in the reduction of his salary. So far, he spent his income lavishly without any planning and prior calculations. However, the prevailing situation compelled him to plan his expenditure to adjust his life to the present income. Therefore, he analysed his spendings of last year i.e., 2019, so that he can plan his present year 2020 spendings. Through his brief analysis, he found that in the year 2019, he spent 23 per cent of his income on food, 15 per cent on Housing, and 12 per cent on his child’s education. For his transport, he spent 10 per cent, and for clothing of his family he spent 10 per cent. In the meanwhile, Mr. Abhishek also saved 15 per cent of his income in the year 2019, and the remaining 20 per cent of expenditure he kept as other spendings. Answer the following questions to provide deeper insight to Mr. Abhishek. 146. If the total amount spent during the year 2019 was 56,000, what was the amount spent on food?

  1. ₹ 12,000
  2. ₹ 12,880
  3. ₹ 23,880
  4. ₹ 23,000.

Answer: (b)

147. If the total amount spent was 56,000, how much money was spent on clothing and housing together?

  1. ₹ 11,500
  2. ₹12,500
  3. ₹14,000
  4. ₹ 15,000.

Answer: (c)

148. What was the ratio of the total amount of money spent on housing to that spent on education?

  1. 5:2
  2. 2:5
  3. 4:5
  4. 5:4.

Answer: (d)

149. If the total expenditure of the family for the year 2019 was 56,000, what was the amount saved by the family during the year?

  1. ₹ 8,400
  2. ₹ 5,600
  3. ₹ 6,900
  4. ₹ 3,067.

Answer: (a)

150. If the total amount spent during the year 2019 was 56,000, the amount spent on other spendings and clothing together was higher than:

  1. Housing and Child Education
  2. Food and Transport
  3. Food and Housing
  4. Housing and Saving.

Answer: (a)

CLAT UG QUESTION PAPER 2021 | Click Here to Download PDF


  1. Law Library: Notes and Study Material for LLB, LLM, Judiciary and Entrance Exams
  2. Legal Bites Academy – Ultimate Test Prep Destination
Updated On 27 Oct 2021 3:08 AM GMT
Admin Legal Bites

Admin Legal Bites

Legal Bites Study Materials correspond to what is taught in law schools and what is tested in competitive exams. It pledges to offer a competitive advantage, prepare for tests, and save a lot of money.

Next Story